Indefinite integral of $frac{tan{(1+x^2)}}{1+x^2}$












6












$begingroup$


Let $f(x) = frac{tan{(1+x^2)}}{1+x^2}$ , find $int f(x)dx$ . I've tried many substitutions (including trigonometric substitutions like $x=tan theta$ ) and also integration by parts but didn't work . We can apply power series but it doesn't solve problem .










share|cite|improve this question









$endgroup$








  • 2




    $begingroup$
    I am afraid that, even using special functions, closed form solutions could be difficult to get.
    $endgroup$
    – Claude Leibovici
    Dec 22 '18 at 8:20






  • 1




    $begingroup$
    Hmm.. this guy is fighting back pretty hard..
    $endgroup$
    – InertialObserver
    Dec 22 '18 at 8:21










  • $begingroup$
    Where did you find this beast?
    $endgroup$
    – Mohammad Zuhair Khan
    Dec 22 '18 at 8:26










  • $begingroup$
    @MohammadZuhairKhan My friend asked that beast!
    $endgroup$
    – S.H.W
    Dec 22 '18 at 8:31






  • 3




    $begingroup$
    Are you sure that he/she is a friend ?
    $endgroup$
    – Claude Leibovici
    Dec 22 '18 at 9:00
















6












$begingroup$


Let $f(x) = frac{tan{(1+x^2)}}{1+x^2}$ , find $int f(x)dx$ . I've tried many substitutions (including trigonometric substitutions like $x=tan theta$ ) and also integration by parts but didn't work . We can apply power series but it doesn't solve problem .










share|cite|improve this question









$endgroup$








  • 2




    $begingroup$
    I am afraid that, even using special functions, closed form solutions could be difficult to get.
    $endgroup$
    – Claude Leibovici
    Dec 22 '18 at 8:20






  • 1




    $begingroup$
    Hmm.. this guy is fighting back pretty hard..
    $endgroup$
    – InertialObserver
    Dec 22 '18 at 8:21










  • $begingroup$
    Where did you find this beast?
    $endgroup$
    – Mohammad Zuhair Khan
    Dec 22 '18 at 8:26










  • $begingroup$
    @MohammadZuhairKhan My friend asked that beast!
    $endgroup$
    – S.H.W
    Dec 22 '18 at 8:31






  • 3




    $begingroup$
    Are you sure that he/she is a friend ?
    $endgroup$
    – Claude Leibovici
    Dec 22 '18 at 9:00














6












6








6


3



$begingroup$


Let $f(x) = frac{tan{(1+x^2)}}{1+x^2}$ , find $int f(x)dx$ . I've tried many substitutions (including trigonometric substitutions like $x=tan theta$ ) and also integration by parts but didn't work . We can apply power series but it doesn't solve problem .










share|cite|improve this question









$endgroup$




Let $f(x) = frac{tan{(1+x^2)}}{1+x^2}$ , find $int f(x)dx$ . I've tried many substitutions (including trigonometric substitutions like $x=tan theta$ ) and also integration by parts but didn't work . We can apply power series but it doesn't solve problem .







real-analysis calculus integration indefinite-integrals






share|cite|improve this question













share|cite|improve this question











share|cite|improve this question




share|cite|improve this question










asked Dec 22 '18 at 8:08









S.H.WS.H.W

1,2031923




1,2031923








  • 2




    $begingroup$
    I am afraid that, even using special functions, closed form solutions could be difficult to get.
    $endgroup$
    – Claude Leibovici
    Dec 22 '18 at 8:20






  • 1




    $begingroup$
    Hmm.. this guy is fighting back pretty hard..
    $endgroup$
    – InertialObserver
    Dec 22 '18 at 8:21










  • $begingroup$
    Where did you find this beast?
    $endgroup$
    – Mohammad Zuhair Khan
    Dec 22 '18 at 8:26










  • $begingroup$
    @MohammadZuhairKhan My friend asked that beast!
    $endgroup$
    – S.H.W
    Dec 22 '18 at 8:31






  • 3




    $begingroup$
    Are you sure that he/she is a friend ?
    $endgroup$
    – Claude Leibovici
    Dec 22 '18 at 9:00














  • 2




    $begingroup$
    I am afraid that, even using special functions, closed form solutions could be difficult to get.
    $endgroup$
    – Claude Leibovici
    Dec 22 '18 at 8:20






  • 1




    $begingroup$
    Hmm.. this guy is fighting back pretty hard..
    $endgroup$
    – InertialObserver
    Dec 22 '18 at 8:21










  • $begingroup$
    Where did you find this beast?
    $endgroup$
    – Mohammad Zuhair Khan
    Dec 22 '18 at 8:26










  • $begingroup$
    @MohammadZuhairKhan My friend asked that beast!
    $endgroup$
    – S.H.W
    Dec 22 '18 at 8:31






  • 3




    $begingroup$
    Are you sure that he/she is a friend ?
    $endgroup$
    – Claude Leibovici
    Dec 22 '18 at 9:00








2




2




$begingroup$
I am afraid that, even using special functions, closed form solutions could be difficult to get.
$endgroup$
– Claude Leibovici
Dec 22 '18 at 8:20




$begingroup$
I am afraid that, even using special functions, closed form solutions could be difficult to get.
$endgroup$
– Claude Leibovici
Dec 22 '18 at 8:20




1




1




$begingroup$
Hmm.. this guy is fighting back pretty hard..
$endgroup$
– InertialObserver
Dec 22 '18 at 8:21




$begingroup$
Hmm.. this guy is fighting back pretty hard..
$endgroup$
– InertialObserver
Dec 22 '18 at 8:21












$begingroup$
Where did you find this beast?
$endgroup$
– Mohammad Zuhair Khan
Dec 22 '18 at 8:26




$begingroup$
Where did you find this beast?
$endgroup$
– Mohammad Zuhair Khan
Dec 22 '18 at 8:26












$begingroup$
@MohammadZuhairKhan My friend asked that beast!
$endgroup$
– S.H.W
Dec 22 '18 at 8:31




$begingroup$
@MohammadZuhairKhan My friend asked that beast!
$endgroup$
– S.H.W
Dec 22 '18 at 8:31




3




3




$begingroup$
Are you sure that he/she is a friend ?
$endgroup$
– Claude Leibovici
Dec 22 '18 at 9:00




$begingroup$
Are you sure that he/she is a friend ?
$endgroup$
– Claude Leibovici
Dec 22 '18 at 9:00










1 Answer
1






active

oldest

votes


















3












$begingroup$

If the problem was
$$int_0^a frac{tan{(1+x^2)}}{1+x^2},dx qquad text{with} qquad 0 leq a lt sqrt{frac{pi -2}{2}}approx 0.7555$$ it could be possible to have an approximation of it using a Padé approximant built at $x=0$.



Using $k=tan(1)$, we should have
$$ frac{tan{(1+x^2)}}{1+x^2}=frac {3 k left(-2 k^2+2 k+1right)+3 left(-k^2+k+1right)x^2 +(2 k-3) left(k^2+1right)x^4} {3(-2 k^2+2 k+1)+ 6 (k-1)^2 k x^2+(6 k^3-7 k^2-4)x^4}$$ which can be integrated using partial fraction decomposition (leading to a nasty expression. Numerically, this would be "almost" (making the coefficient rational)
$$int frac{tan{(1+x^2)}}{1+x^2},dx simeq frac{184 }{789}x+frac{1099} {3526}tan ^{-1}left(frac{529
}{801}xright)+frac{1409 }{1667}tanh ^{-1}left(frac{692
}{523}xright)$$
For a few values of $a$, some results (the so called "exact" being obtained by numerical integration)
$$left(
begin{array}{ccc}
a & text{approximation} & text{exact} \
0.05 & 0.07795 & 0.07795 \
0.10 & 0.15637 & 0.15637 \
0.15 & 0.23577 & 0.23577 \
0.20 & 0.31670 & 0.31670 \
0.25 & 0.39982 & 0.39982 \
0.30 & 0.48593 & 0.48593 \
0.35 & 0.57609 & 0.57610 \
0.40 & 0.67171 & 0.67172 \
0.45 & 0.77482 & 0.77482 \
0.50 & 0.88848 & 0.88848 \
0.55 & 1.01775 & 1.01775 \
0.60 & 1.17192 & 1.17195 \
0.65 & 1.37127 & 1.37136 \
0.70 & 1.67673 & 1.67722 \
0.75 & 2.66909 & 2.68502
end{array}
right)$$



Edit



We can make the approximation better using the fact that
$$f(x)=left(x^2-frac{pi }{2}+1right)frac{ tan left(x^2+1right)}{x^2+1}$$ is a quite nice function. Using a $[4,2]$ Padé approximant, we get
$$f(x) simeqfrac {a_0+a_1 x^2+a_2 x^4 }{b_0+b_1 x^2 }$$ where
$$a_0=3 (pi -2) k left(pi (k ((k-1) k+2)-1)-2 left(k^3+kright)right)$$
$$a_1=6 pi -left(k^2+1right) left(3 (pi -2) pi k^2+((8-5 pi ) pi -8) k+3 pi ^2right)$$
$$a_2=left(k^2+1right) left(2 (pi -2) (1+pi ) k^2-2 pi (1+pi ) k-(pi -10) pi -4right)$$
$$b_0=6 (k (k (pi -(pi -2) k)-2 pi +2)+pi )$$
$$b_1=2 left(k left(k left(3 (pi -2) k^2-3 pi k+7 pi -8right)-6 pi right)+4 pi -2right)$$ leading to
$$int frac{tan{(1+x^2)}}{1+x^2},dx simeq frac{a_2 }{b_1}x-frac{2 left(b_1 (a_0 b_1-a_1 b_0)+a_2
b_0^2right)}{sqrt{b_0} b_1^{3/2} (2 b_0+(pi -2) b_1)}tan ^{-1}left(frac{sqrt{b_1} }{sqrt{b_0}}xright) -frac{(4 a_0+(pi -2) (2 a_1+(pi -2)
a_2))}{sqrt{2 (pi -2)} (2 b_0+(pi -2) b_1)}tanh
^{-1}left(sqrt{frac{2}{pi -2}} xright) $$
Making the same table as before
$$left(
begin{array}{ccc}
a & text{approximation} & text{exact} \
0.05 & 0.07795 & 0.07795 \
0.10 & 0.15637 & 0.15637 \
0.15 & 0.23577 & 0.23577 \
0.20 & 0.31670 & 0.31670 \
0.25 & 0.39982 & 0.39982 \
0.30 & 0.48593 & 0.48593 \
0.35 & 0.57610 & 0.57610 \
0.40 & 0.67172 & 0.67172 \
0.45 & 0.77482 & 0.77482 \
0.50 & 0.88848 & 0.88848 \
0.55 & 1.01776 & 1.01775 \
0.60 & 1.17197 & 1.17195 \
0.65 & 1.37142 & 1.37136 \
0.70 & 1.67740 & 1.67722 \
0.75 & 2.68589 & 2.68502
end{array}
right)$$
which looks quite better.






share|cite|improve this answer











$endgroup$













  • $begingroup$
    Thanks , I keep this question open for more answers .
    $endgroup$
    – S.H.W
    Dec 22 '18 at 18:27










  • $begingroup$
    @S.H.W : For sure, keep it open. It is very interesting. I am still working on it. Cheers.
    $endgroup$
    – Claude Leibovici
    Dec 23 '18 at 2:34










  • $begingroup$
    @Claude Leibovici, I think my Maxima made an error , soon I shall post a better answer.
    $endgroup$
    – Awe Kumar Jha
    Dec 24 '18 at 12:38











Your Answer





StackExchange.ifUsing("editor", function () {
return StackExchange.using("mathjaxEditing", function () {
StackExchange.MarkdownEditor.creationCallbacks.add(function (editor, postfix) {
StackExchange.mathjaxEditing.prepareWmdForMathJax(editor, postfix, [["$", "$"], ["\\(","\\)"]]);
});
});
}, "mathjax-editing");

StackExchange.ready(function() {
var channelOptions = {
tags: "".split(" "),
id: "69"
};
initTagRenderer("".split(" "), "".split(" "), channelOptions);

StackExchange.using("externalEditor", function() {
// Have to fire editor after snippets, if snippets enabled
if (StackExchange.settings.snippets.snippetsEnabled) {
StackExchange.using("snippets", function() {
createEditor();
});
}
else {
createEditor();
}
});

function createEditor() {
StackExchange.prepareEditor({
heartbeatType: 'answer',
autoActivateHeartbeat: false,
convertImagesToLinks: true,
noModals: true,
showLowRepImageUploadWarning: true,
reputationToPostImages: 10,
bindNavPrevention: true,
postfix: "",
imageUploader: {
brandingHtml: "Powered by u003ca class="icon-imgur-white" href="https://imgur.com/"u003eu003c/au003e",
contentPolicyHtml: "User contributions licensed under u003ca href="https://creativecommons.org/licenses/by-sa/3.0/"u003ecc by-sa 3.0 with attribution requiredu003c/au003e u003ca href="https://stackoverflow.com/legal/content-policy"u003e(content policy)u003c/au003e",
allowUrls: true
},
noCode: true, onDemand: true,
discardSelector: ".discard-answer"
,immediatelyShowMarkdownHelp:true
});


}
});














draft saved

draft discarded


















StackExchange.ready(
function () {
StackExchange.openid.initPostLogin('.new-post-login', 'https%3a%2f%2fmath.stackexchange.com%2fquestions%2f3049210%2findefinite-integral-of-frac-tan1x21x2%23new-answer', 'question_page');
}
);

Post as a guest















Required, but never shown

























1 Answer
1






active

oldest

votes








1 Answer
1






active

oldest

votes









active

oldest

votes






active

oldest

votes









3












$begingroup$

If the problem was
$$int_0^a frac{tan{(1+x^2)}}{1+x^2},dx qquad text{with} qquad 0 leq a lt sqrt{frac{pi -2}{2}}approx 0.7555$$ it could be possible to have an approximation of it using a Padé approximant built at $x=0$.



Using $k=tan(1)$, we should have
$$ frac{tan{(1+x^2)}}{1+x^2}=frac {3 k left(-2 k^2+2 k+1right)+3 left(-k^2+k+1right)x^2 +(2 k-3) left(k^2+1right)x^4} {3(-2 k^2+2 k+1)+ 6 (k-1)^2 k x^2+(6 k^3-7 k^2-4)x^4}$$ which can be integrated using partial fraction decomposition (leading to a nasty expression. Numerically, this would be "almost" (making the coefficient rational)
$$int frac{tan{(1+x^2)}}{1+x^2},dx simeq frac{184 }{789}x+frac{1099} {3526}tan ^{-1}left(frac{529
}{801}xright)+frac{1409 }{1667}tanh ^{-1}left(frac{692
}{523}xright)$$
For a few values of $a$, some results (the so called "exact" being obtained by numerical integration)
$$left(
begin{array}{ccc}
a & text{approximation} & text{exact} \
0.05 & 0.07795 & 0.07795 \
0.10 & 0.15637 & 0.15637 \
0.15 & 0.23577 & 0.23577 \
0.20 & 0.31670 & 0.31670 \
0.25 & 0.39982 & 0.39982 \
0.30 & 0.48593 & 0.48593 \
0.35 & 0.57609 & 0.57610 \
0.40 & 0.67171 & 0.67172 \
0.45 & 0.77482 & 0.77482 \
0.50 & 0.88848 & 0.88848 \
0.55 & 1.01775 & 1.01775 \
0.60 & 1.17192 & 1.17195 \
0.65 & 1.37127 & 1.37136 \
0.70 & 1.67673 & 1.67722 \
0.75 & 2.66909 & 2.68502
end{array}
right)$$



Edit



We can make the approximation better using the fact that
$$f(x)=left(x^2-frac{pi }{2}+1right)frac{ tan left(x^2+1right)}{x^2+1}$$ is a quite nice function. Using a $[4,2]$ Padé approximant, we get
$$f(x) simeqfrac {a_0+a_1 x^2+a_2 x^4 }{b_0+b_1 x^2 }$$ where
$$a_0=3 (pi -2) k left(pi (k ((k-1) k+2)-1)-2 left(k^3+kright)right)$$
$$a_1=6 pi -left(k^2+1right) left(3 (pi -2) pi k^2+((8-5 pi ) pi -8) k+3 pi ^2right)$$
$$a_2=left(k^2+1right) left(2 (pi -2) (1+pi ) k^2-2 pi (1+pi ) k-(pi -10) pi -4right)$$
$$b_0=6 (k (k (pi -(pi -2) k)-2 pi +2)+pi )$$
$$b_1=2 left(k left(k left(3 (pi -2) k^2-3 pi k+7 pi -8right)-6 pi right)+4 pi -2right)$$ leading to
$$int frac{tan{(1+x^2)}}{1+x^2},dx simeq frac{a_2 }{b_1}x-frac{2 left(b_1 (a_0 b_1-a_1 b_0)+a_2
b_0^2right)}{sqrt{b_0} b_1^{3/2} (2 b_0+(pi -2) b_1)}tan ^{-1}left(frac{sqrt{b_1} }{sqrt{b_0}}xright) -frac{(4 a_0+(pi -2) (2 a_1+(pi -2)
a_2))}{sqrt{2 (pi -2)} (2 b_0+(pi -2) b_1)}tanh
^{-1}left(sqrt{frac{2}{pi -2}} xright) $$
Making the same table as before
$$left(
begin{array}{ccc}
a & text{approximation} & text{exact} \
0.05 & 0.07795 & 0.07795 \
0.10 & 0.15637 & 0.15637 \
0.15 & 0.23577 & 0.23577 \
0.20 & 0.31670 & 0.31670 \
0.25 & 0.39982 & 0.39982 \
0.30 & 0.48593 & 0.48593 \
0.35 & 0.57610 & 0.57610 \
0.40 & 0.67172 & 0.67172 \
0.45 & 0.77482 & 0.77482 \
0.50 & 0.88848 & 0.88848 \
0.55 & 1.01776 & 1.01775 \
0.60 & 1.17197 & 1.17195 \
0.65 & 1.37142 & 1.37136 \
0.70 & 1.67740 & 1.67722 \
0.75 & 2.68589 & 2.68502
end{array}
right)$$
which looks quite better.






share|cite|improve this answer











$endgroup$













  • $begingroup$
    Thanks , I keep this question open for more answers .
    $endgroup$
    – S.H.W
    Dec 22 '18 at 18:27










  • $begingroup$
    @S.H.W : For sure, keep it open. It is very interesting. I am still working on it. Cheers.
    $endgroup$
    – Claude Leibovici
    Dec 23 '18 at 2:34










  • $begingroup$
    @Claude Leibovici, I think my Maxima made an error , soon I shall post a better answer.
    $endgroup$
    – Awe Kumar Jha
    Dec 24 '18 at 12:38
















3












$begingroup$

If the problem was
$$int_0^a frac{tan{(1+x^2)}}{1+x^2},dx qquad text{with} qquad 0 leq a lt sqrt{frac{pi -2}{2}}approx 0.7555$$ it could be possible to have an approximation of it using a Padé approximant built at $x=0$.



Using $k=tan(1)$, we should have
$$ frac{tan{(1+x^2)}}{1+x^2}=frac {3 k left(-2 k^2+2 k+1right)+3 left(-k^2+k+1right)x^2 +(2 k-3) left(k^2+1right)x^4} {3(-2 k^2+2 k+1)+ 6 (k-1)^2 k x^2+(6 k^3-7 k^2-4)x^4}$$ which can be integrated using partial fraction decomposition (leading to a nasty expression. Numerically, this would be "almost" (making the coefficient rational)
$$int frac{tan{(1+x^2)}}{1+x^2},dx simeq frac{184 }{789}x+frac{1099} {3526}tan ^{-1}left(frac{529
}{801}xright)+frac{1409 }{1667}tanh ^{-1}left(frac{692
}{523}xright)$$
For a few values of $a$, some results (the so called "exact" being obtained by numerical integration)
$$left(
begin{array}{ccc}
a & text{approximation} & text{exact} \
0.05 & 0.07795 & 0.07795 \
0.10 & 0.15637 & 0.15637 \
0.15 & 0.23577 & 0.23577 \
0.20 & 0.31670 & 0.31670 \
0.25 & 0.39982 & 0.39982 \
0.30 & 0.48593 & 0.48593 \
0.35 & 0.57609 & 0.57610 \
0.40 & 0.67171 & 0.67172 \
0.45 & 0.77482 & 0.77482 \
0.50 & 0.88848 & 0.88848 \
0.55 & 1.01775 & 1.01775 \
0.60 & 1.17192 & 1.17195 \
0.65 & 1.37127 & 1.37136 \
0.70 & 1.67673 & 1.67722 \
0.75 & 2.66909 & 2.68502
end{array}
right)$$



Edit



We can make the approximation better using the fact that
$$f(x)=left(x^2-frac{pi }{2}+1right)frac{ tan left(x^2+1right)}{x^2+1}$$ is a quite nice function. Using a $[4,2]$ Padé approximant, we get
$$f(x) simeqfrac {a_0+a_1 x^2+a_2 x^4 }{b_0+b_1 x^2 }$$ where
$$a_0=3 (pi -2) k left(pi (k ((k-1) k+2)-1)-2 left(k^3+kright)right)$$
$$a_1=6 pi -left(k^2+1right) left(3 (pi -2) pi k^2+((8-5 pi ) pi -8) k+3 pi ^2right)$$
$$a_2=left(k^2+1right) left(2 (pi -2) (1+pi ) k^2-2 pi (1+pi ) k-(pi -10) pi -4right)$$
$$b_0=6 (k (k (pi -(pi -2) k)-2 pi +2)+pi )$$
$$b_1=2 left(k left(k left(3 (pi -2) k^2-3 pi k+7 pi -8right)-6 pi right)+4 pi -2right)$$ leading to
$$int frac{tan{(1+x^2)}}{1+x^2},dx simeq frac{a_2 }{b_1}x-frac{2 left(b_1 (a_0 b_1-a_1 b_0)+a_2
b_0^2right)}{sqrt{b_0} b_1^{3/2} (2 b_0+(pi -2) b_1)}tan ^{-1}left(frac{sqrt{b_1} }{sqrt{b_0}}xright) -frac{(4 a_0+(pi -2) (2 a_1+(pi -2)
a_2))}{sqrt{2 (pi -2)} (2 b_0+(pi -2) b_1)}tanh
^{-1}left(sqrt{frac{2}{pi -2}} xright) $$
Making the same table as before
$$left(
begin{array}{ccc}
a & text{approximation} & text{exact} \
0.05 & 0.07795 & 0.07795 \
0.10 & 0.15637 & 0.15637 \
0.15 & 0.23577 & 0.23577 \
0.20 & 0.31670 & 0.31670 \
0.25 & 0.39982 & 0.39982 \
0.30 & 0.48593 & 0.48593 \
0.35 & 0.57610 & 0.57610 \
0.40 & 0.67172 & 0.67172 \
0.45 & 0.77482 & 0.77482 \
0.50 & 0.88848 & 0.88848 \
0.55 & 1.01776 & 1.01775 \
0.60 & 1.17197 & 1.17195 \
0.65 & 1.37142 & 1.37136 \
0.70 & 1.67740 & 1.67722 \
0.75 & 2.68589 & 2.68502
end{array}
right)$$
which looks quite better.






share|cite|improve this answer











$endgroup$













  • $begingroup$
    Thanks , I keep this question open for more answers .
    $endgroup$
    – S.H.W
    Dec 22 '18 at 18:27










  • $begingroup$
    @S.H.W : For sure, keep it open. It is very interesting. I am still working on it. Cheers.
    $endgroup$
    – Claude Leibovici
    Dec 23 '18 at 2:34










  • $begingroup$
    @Claude Leibovici, I think my Maxima made an error , soon I shall post a better answer.
    $endgroup$
    – Awe Kumar Jha
    Dec 24 '18 at 12:38














3












3








3





$begingroup$

If the problem was
$$int_0^a frac{tan{(1+x^2)}}{1+x^2},dx qquad text{with} qquad 0 leq a lt sqrt{frac{pi -2}{2}}approx 0.7555$$ it could be possible to have an approximation of it using a Padé approximant built at $x=0$.



Using $k=tan(1)$, we should have
$$ frac{tan{(1+x^2)}}{1+x^2}=frac {3 k left(-2 k^2+2 k+1right)+3 left(-k^2+k+1right)x^2 +(2 k-3) left(k^2+1right)x^4} {3(-2 k^2+2 k+1)+ 6 (k-1)^2 k x^2+(6 k^3-7 k^2-4)x^4}$$ which can be integrated using partial fraction decomposition (leading to a nasty expression. Numerically, this would be "almost" (making the coefficient rational)
$$int frac{tan{(1+x^2)}}{1+x^2},dx simeq frac{184 }{789}x+frac{1099} {3526}tan ^{-1}left(frac{529
}{801}xright)+frac{1409 }{1667}tanh ^{-1}left(frac{692
}{523}xright)$$
For a few values of $a$, some results (the so called "exact" being obtained by numerical integration)
$$left(
begin{array}{ccc}
a & text{approximation} & text{exact} \
0.05 & 0.07795 & 0.07795 \
0.10 & 0.15637 & 0.15637 \
0.15 & 0.23577 & 0.23577 \
0.20 & 0.31670 & 0.31670 \
0.25 & 0.39982 & 0.39982 \
0.30 & 0.48593 & 0.48593 \
0.35 & 0.57609 & 0.57610 \
0.40 & 0.67171 & 0.67172 \
0.45 & 0.77482 & 0.77482 \
0.50 & 0.88848 & 0.88848 \
0.55 & 1.01775 & 1.01775 \
0.60 & 1.17192 & 1.17195 \
0.65 & 1.37127 & 1.37136 \
0.70 & 1.67673 & 1.67722 \
0.75 & 2.66909 & 2.68502
end{array}
right)$$



Edit



We can make the approximation better using the fact that
$$f(x)=left(x^2-frac{pi }{2}+1right)frac{ tan left(x^2+1right)}{x^2+1}$$ is a quite nice function. Using a $[4,2]$ Padé approximant, we get
$$f(x) simeqfrac {a_0+a_1 x^2+a_2 x^4 }{b_0+b_1 x^2 }$$ where
$$a_0=3 (pi -2) k left(pi (k ((k-1) k+2)-1)-2 left(k^3+kright)right)$$
$$a_1=6 pi -left(k^2+1right) left(3 (pi -2) pi k^2+((8-5 pi ) pi -8) k+3 pi ^2right)$$
$$a_2=left(k^2+1right) left(2 (pi -2) (1+pi ) k^2-2 pi (1+pi ) k-(pi -10) pi -4right)$$
$$b_0=6 (k (k (pi -(pi -2) k)-2 pi +2)+pi )$$
$$b_1=2 left(k left(k left(3 (pi -2) k^2-3 pi k+7 pi -8right)-6 pi right)+4 pi -2right)$$ leading to
$$int frac{tan{(1+x^2)}}{1+x^2},dx simeq frac{a_2 }{b_1}x-frac{2 left(b_1 (a_0 b_1-a_1 b_0)+a_2
b_0^2right)}{sqrt{b_0} b_1^{3/2} (2 b_0+(pi -2) b_1)}tan ^{-1}left(frac{sqrt{b_1} }{sqrt{b_0}}xright) -frac{(4 a_0+(pi -2) (2 a_1+(pi -2)
a_2))}{sqrt{2 (pi -2)} (2 b_0+(pi -2) b_1)}tanh
^{-1}left(sqrt{frac{2}{pi -2}} xright) $$
Making the same table as before
$$left(
begin{array}{ccc}
a & text{approximation} & text{exact} \
0.05 & 0.07795 & 0.07795 \
0.10 & 0.15637 & 0.15637 \
0.15 & 0.23577 & 0.23577 \
0.20 & 0.31670 & 0.31670 \
0.25 & 0.39982 & 0.39982 \
0.30 & 0.48593 & 0.48593 \
0.35 & 0.57610 & 0.57610 \
0.40 & 0.67172 & 0.67172 \
0.45 & 0.77482 & 0.77482 \
0.50 & 0.88848 & 0.88848 \
0.55 & 1.01776 & 1.01775 \
0.60 & 1.17197 & 1.17195 \
0.65 & 1.37142 & 1.37136 \
0.70 & 1.67740 & 1.67722 \
0.75 & 2.68589 & 2.68502
end{array}
right)$$
which looks quite better.






share|cite|improve this answer











$endgroup$



If the problem was
$$int_0^a frac{tan{(1+x^2)}}{1+x^2},dx qquad text{with} qquad 0 leq a lt sqrt{frac{pi -2}{2}}approx 0.7555$$ it could be possible to have an approximation of it using a Padé approximant built at $x=0$.



Using $k=tan(1)$, we should have
$$ frac{tan{(1+x^2)}}{1+x^2}=frac {3 k left(-2 k^2+2 k+1right)+3 left(-k^2+k+1right)x^2 +(2 k-3) left(k^2+1right)x^4} {3(-2 k^2+2 k+1)+ 6 (k-1)^2 k x^2+(6 k^3-7 k^2-4)x^4}$$ which can be integrated using partial fraction decomposition (leading to a nasty expression. Numerically, this would be "almost" (making the coefficient rational)
$$int frac{tan{(1+x^2)}}{1+x^2},dx simeq frac{184 }{789}x+frac{1099} {3526}tan ^{-1}left(frac{529
}{801}xright)+frac{1409 }{1667}tanh ^{-1}left(frac{692
}{523}xright)$$
For a few values of $a$, some results (the so called "exact" being obtained by numerical integration)
$$left(
begin{array}{ccc}
a & text{approximation} & text{exact} \
0.05 & 0.07795 & 0.07795 \
0.10 & 0.15637 & 0.15637 \
0.15 & 0.23577 & 0.23577 \
0.20 & 0.31670 & 0.31670 \
0.25 & 0.39982 & 0.39982 \
0.30 & 0.48593 & 0.48593 \
0.35 & 0.57609 & 0.57610 \
0.40 & 0.67171 & 0.67172 \
0.45 & 0.77482 & 0.77482 \
0.50 & 0.88848 & 0.88848 \
0.55 & 1.01775 & 1.01775 \
0.60 & 1.17192 & 1.17195 \
0.65 & 1.37127 & 1.37136 \
0.70 & 1.67673 & 1.67722 \
0.75 & 2.66909 & 2.68502
end{array}
right)$$



Edit



We can make the approximation better using the fact that
$$f(x)=left(x^2-frac{pi }{2}+1right)frac{ tan left(x^2+1right)}{x^2+1}$$ is a quite nice function. Using a $[4,2]$ Padé approximant, we get
$$f(x) simeqfrac {a_0+a_1 x^2+a_2 x^4 }{b_0+b_1 x^2 }$$ where
$$a_0=3 (pi -2) k left(pi (k ((k-1) k+2)-1)-2 left(k^3+kright)right)$$
$$a_1=6 pi -left(k^2+1right) left(3 (pi -2) pi k^2+((8-5 pi ) pi -8) k+3 pi ^2right)$$
$$a_2=left(k^2+1right) left(2 (pi -2) (1+pi ) k^2-2 pi (1+pi ) k-(pi -10) pi -4right)$$
$$b_0=6 (k (k (pi -(pi -2) k)-2 pi +2)+pi )$$
$$b_1=2 left(k left(k left(3 (pi -2) k^2-3 pi k+7 pi -8right)-6 pi right)+4 pi -2right)$$ leading to
$$int frac{tan{(1+x^2)}}{1+x^2},dx simeq frac{a_2 }{b_1}x-frac{2 left(b_1 (a_0 b_1-a_1 b_0)+a_2
b_0^2right)}{sqrt{b_0} b_1^{3/2} (2 b_0+(pi -2) b_1)}tan ^{-1}left(frac{sqrt{b_1} }{sqrt{b_0}}xright) -frac{(4 a_0+(pi -2) (2 a_1+(pi -2)
a_2))}{sqrt{2 (pi -2)} (2 b_0+(pi -2) b_1)}tanh
^{-1}left(sqrt{frac{2}{pi -2}} xright) $$
Making the same table as before
$$left(
begin{array}{ccc}
a & text{approximation} & text{exact} \
0.05 & 0.07795 & 0.07795 \
0.10 & 0.15637 & 0.15637 \
0.15 & 0.23577 & 0.23577 \
0.20 & 0.31670 & 0.31670 \
0.25 & 0.39982 & 0.39982 \
0.30 & 0.48593 & 0.48593 \
0.35 & 0.57610 & 0.57610 \
0.40 & 0.67172 & 0.67172 \
0.45 & 0.77482 & 0.77482 \
0.50 & 0.88848 & 0.88848 \
0.55 & 1.01776 & 1.01775 \
0.60 & 1.17197 & 1.17195 \
0.65 & 1.37142 & 1.37136 \
0.70 & 1.67740 & 1.67722 \
0.75 & 2.68589 & 2.68502
end{array}
right)$$
which looks quite better.







share|cite|improve this answer














share|cite|improve this answer



share|cite|improve this answer








edited Dec 22 '18 at 14:39

























answered Dec 22 '18 at 9:49









Claude LeiboviciClaude Leibovici

122k1157134




122k1157134












  • $begingroup$
    Thanks , I keep this question open for more answers .
    $endgroup$
    – S.H.W
    Dec 22 '18 at 18:27










  • $begingroup$
    @S.H.W : For sure, keep it open. It is very interesting. I am still working on it. Cheers.
    $endgroup$
    – Claude Leibovici
    Dec 23 '18 at 2:34










  • $begingroup$
    @Claude Leibovici, I think my Maxima made an error , soon I shall post a better answer.
    $endgroup$
    – Awe Kumar Jha
    Dec 24 '18 at 12:38


















  • $begingroup$
    Thanks , I keep this question open for more answers .
    $endgroup$
    – S.H.W
    Dec 22 '18 at 18:27










  • $begingroup$
    @S.H.W : For sure, keep it open. It is very interesting. I am still working on it. Cheers.
    $endgroup$
    – Claude Leibovici
    Dec 23 '18 at 2:34










  • $begingroup$
    @Claude Leibovici, I think my Maxima made an error , soon I shall post a better answer.
    $endgroup$
    – Awe Kumar Jha
    Dec 24 '18 at 12:38
















$begingroup$
Thanks , I keep this question open for more answers .
$endgroup$
– S.H.W
Dec 22 '18 at 18:27




$begingroup$
Thanks , I keep this question open for more answers .
$endgroup$
– S.H.W
Dec 22 '18 at 18:27












$begingroup$
@S.H.W : For sure, keep it open. It is very interesting. I am still working on it. Cheers.
$endgroup$
– Claude Leibovici
Dec 23 '18 at 2:34




$begingroup$
@S.H.W : For sure, keep it open. It is very interesting. I am still working on it. Cheers.
$endgroup$
– Claude Leibovici
Dec 23 '18 at 2:34












$begingroup$
@Claude Leibovici, I think my Maxima made an error , soon I shall post a better answer.
$endgroup$
– Awe Kumar Jha
Dec 24 '18 at 12:38




$begingroup$
@Claude Leibovici, I think my Maxima made an error , soon I shall post a better answer.
$endgroup$
– Awe Kumar Jha
Dec 24 '18 at 12:38


















draft saved

draft discarded




















































Thanks for contributing an answer to Mathematics Stack Exchange!


  • Please be sure to answer the question. Provide details and share your research!

But avoid



  • Asking for help, clarification, or responding to other answers.

  • Making statements based on opinion; back them up with references or personal experience.


Use MathJax to format equations. MathJax reference.


To learn more, see our tips on writing great answers.




draft saved


draft discarded














StackExchange.ready(
function () {
StackExchange.openid.initPostLogin('.new-post-login', 'https%3a%2f%2fmath.stackexchange.com%2fquestions%2f3049210%2findefinite-integral-of-frac-tan1x21x2%23new-answer', 'question_page');
}
);

Post as a guest















Required, but never shown





















































Required, but never shown














Required, but never shown












Required, but never shown







Required, but never shown

































Required, but never shown














Required, but never shown












Required, but never shown







Required, but never shown







Popular posts from this blog

Quarter-circle Tiles

build a pushdown automaton that recognizes the reverse language of a given pushdown automaton?

Mont Emei